Search found 18 matches


GMAT prep - a couple more

I appreciate all the help I have got on this forum.
A couple more questions to solve.

by rajt

Mon May 21, 2007 9:20 am
Forum: Data Sufficiency
Topic: GMAT prep - a couple more
Replies: 1
Views: 1855

GMAT prep

How do I solve this?

by rajt

Sun May 20, 2007 6:43 pm
Forum: Problem Solving
Topic: GMAT prep
Replies: 3
Views: 2868

GMAT prep

I got these wrong. Can you please help me solve them? Thanks.

by rajt

Sun May 20, 2007 6:41 pm
Forum: Data Sufficiency
Topic: GMAT prep
Replies: 3
Views: 2447

10s digit of k + 5 = 4. How do we represent this as a formula?

Say k = 10 * t + 1 * u

10*t + 5 = 4

by rajt

Sun May 20, 2007 12:30 pm
Forum: Data Sufficiency
Topic: Help!!
Replies: 3
Views: 2506

1. Insufficient
2. Insufficient

from 1, n-3= 2k
n-3-4 = 2k - 4 (add -4 on either side)
=> n= (2k-4) + 7
from (2) 2k-4 is divisible by 7 and 7 divides 7 1 time.

So C.

by rajt

Sun May 20, 2007 11:47 am
Forum: Data Sufficiency
Topic: divisibility question
Replies: 5
Views: 2782

Ok, now I understand. Thank you.
Raj

by rajt

Sun May 20, 2007 11:21 am
Forum: Data Sufficiency
Topic: GMAT prep
Replies: 4
Views: 2577

here is what I do not understand. Statement 2 states that the number of integers are even which means the set may consist of {1, 2, -3, 4} or {-1,-2,3,4} or {1, -2, 3, 4, 5, 6} as long as the number of numbers are positive. using 1 and the set {1,2,-3,4} product of 1 and 4 = positive (product of low...

by rajt

Sun May 20, 2007 10:59 am
Forum: Data Sufficiency
Topic: GMAT prep
Replies: 4
Views: 2577

Help!!

How do I solve these?

by rajt

Sun May 20, 2007 10:48 am
Forum: Data Sufficiency
Topic: Help!!
Replies: 3
Views: 2506

what is the quicket way to solve this?

Is there a quick way to solve this, instead of substituting values?

by rajt

Sun May 20, 2007 10:28 am
Forum: Data Sufficiency
Topic: what is the quicket way to solve this?
Replies: 2
Views: 2244

GMAT prep

Please help me solve this. Thanks

by rajt

Sun May 20, 2007 10:18 am
Forum: Data Sufficiency
Topic: GMAT prep
Replies: 4
Views: 2577

GMAT prep

Is my reasoning correct?
1. Insufficient; sum of all terms = 3124
2. Insufficient; Sum of all terms/n= 4

Combining 1 and 2 we get 3124/n=4, so n=3124/4, hence C

by rajt

Sat May 19, 2007 11:29 am
Forum: Data Sufficiency
Topic: GMAT prep
Replies: 1
Views: 2096

GMAT prep

If the operation D is one of the four arithmetic operations addition, subtraction, multiplication and division, is (6 D 2) D 4 = 6 D (2 D 4) 1. 3 D 2 >3 2. 3 D 1 = 3 Ans = A I am getting C and here is how I have solved it From 1 D can be either + or X (because 3 +2 >3 and 3 X 2>3) - not sufficient F...

by rajt

Sat May 19, 2007 10:52 am
Forum: Data Sufficiency
Topic: GMAT prep
Replies: 1
Views: 2722

GMA prep - help

Can you please help me solve this?

by rajt

Sat May 19, 2007 10:00 am
Forum: Problem Solving
Topic: GMA prep - help
Replies: 2
Views: 2787

yes the answer is D. Your explanation is pretty clear.

by rajt

Fri May 18, 2007 7:22 pm
Forum: Data Sufficiency
Topic: I need help
Replies: 2
Views: 2281

6 is teh right answer. Thanks Jay!

by rajt

Fri May 18, 2007 4:05 pm
Forum: Problem Solving
Topic: GMAT prep
Replies: 6
Views: 3168